Selma uses a jogging trail that runs through a park near her home. The trail is a loop that is of a mile long. On Monday, Selma ran the loop in of an hour. What is Selma’s unit rate in miles per hour for Monday’s run?


In this activity, you will use the common denominator method to calculate a unit rate that involves fractions. Answer the questions that follow to calculate Selma’s unit rate in miles per hour.


part A. According to the question, the unit rate is to be expressed in which units?


Part B. Now, write Selma’s jogging rate as a complex fraction. A complex fraction is one whose numerator, denominator, or both are fractions. Be sure to include units in your answer.


Part C. Find the least common denominator of the two fractions from part B. To do this, list and compare the first five multiples of the denominators. The smallest number that appears in both lists is the least common denominator. Show your work.


Part D

Answers

Answer 1

Answer:

a) Her rate is 1 mile per hour  for Monday’s run

b) 0.447 m/second

c) In fraction  20467m/180000 seconds

Step-by-step explanation:

Speed = distance / time = 1 mile/ 1 hour  

           = 1 mile per hour

Her rate is 1 mile per hour  for Monday’s run

part A. According to the question, the unit rate is to be expressed in miles per hour units.

Part B.

Speed = distance / time = 1 mile/ 1 hour  

           = 1 mile per hour

But 1 mile = 1609.34 m

and 1 hour = 3600 seconds

There fore  

1 mile/ 1 hour  = 1*1609.34/ 1*3600 seconds

                    = 0.447 m/second

Part C.

1 mile/ 1 hour  =1609.34/ 3600 seconds

                     = 1609.34 * 100/100 * 1/3600  ( multiplying and dividing by 100 to remove the decimal)

                     = 160934/360000

                      = 20467m/180000 seconds

                       

The first five multiples of 1 are 2,3,4,5,6,7,8, 9......

The LCM of 1 and 1 is 1


Related Questions

The equation below describes a proportional relationship between x and y. What is the constant of​ proportionality? y=3/5x The constant of proportionality is______

Answers

Answer:inversely proportional

Step-by-step explanation:

As the value of x increase the value of y decrease

Solve four and three fourths minus one and one third.

(GIVING EXTRA POINTS ON THIS ONE AND A BREAINLIEST!!)

A) Two and two sevenths

B) two and two twelfths

C) three and five twelfths

D) three and five sevenths

(Here is an image if needed) :

Answers

Answer: c.

Explanation: The first thing you do when you subtract mixed numbers is convert them into improper fractions. 4 3/4 can be converted to 19/4 and 1 1/3 can be converted into 4/3. To be able to subtract or add fractions they need to have the same denominator a number in which both denominators can go into and that number is 12. 4 can equally go into 12 and so can 3. Now that you have found the common denominator (12) you have to find out what kind of operation you need to do in order for 4 to be 12 and for 3 to also be 12. And that is by multiplying. If you multiply 4 by 3 and get 12 and 3 by 4 to get 12 as well. Whatever you do to the bottom of the fraction you also do to the top so 19 x 3 / 4 x 3 = 57/12. Do the same to the other improper fraction: 4 x 4/ 3 x4 = 16/12. They both now have the same denominator but you have to subtract so,,57/12 - 16/12 = 41/12 you have the answer to the subtraction problem but you can probably tell that it isn’t any of the options on the question. Simplifying 41/12 will give you the mixed number 3 5/12.

Answer:

c

Step-by-step explanation:

The cost of medical care abroad is often significantly less than in the United States. In 2014, the cost of knee replacement surgery in the United states was $34,000. This cost was $4000 more than four times the cost of knee replacement surgery in India. Find the cost of Knee replacement surgery in India.

Answers

Answer:

Cost in India: $7500

Step-by-step explanation:

Represent cost in India with I and cost in United States as U.

Given

[tex]U = 3400[/tex]

[tex]U = 4000 + 4 * I[/tex]

Required

Find I

Simply substitute 34000 for U in the second equation

[tex]34000 = 4000 + 4 * I[/tex]

[tex]34000 = 4000 + 4I[/tex]

Solve for 4I

[tex]4I = 34000 - 4000[/tex]

[tex]4I = 30000[/tex]

Solve for I

[tex]I = 30000/4[/tex]

[tex]I = 7500[/tex]

Hence, the cost in India is $7500

Which of the following rates are proportional to $3.85 per 1 pound? Select all that apply.
HELP STAT
A) 5 pounds for $19.25
B) $7.70 for 2 pounds
C) 4 pounds for $15.10
D) $10.00 for 3 pounds
E) 10 pounds for $38.50
F) $28.75 for 9 pounds

Answers

U just divide the money by how many pound and if u get 3.85 that is correct
A, B, and E
A:$19.25/5=$3.85
B:$7.70/2=$3.85
E:$38.50/10=$3.85

A newborn giraffe weighs about 65 kilograms. How much does it weigh in grams?

Answers

Answer:

65,000 grams

Step-by-step explanation:

1 kilogram = 1,000 grams

The giraffe weighs 65,000 grams.

A 2-column table with 6 rows. Column 1 is labeled Tables with entries 1, 2, 3, 4, 5, 6. Column 2 is labeled Chairs with entries 6, 12, 18, 24, 30, 36.
Rachel needs two tables for her birthday party. How many chairs will she need?

Answers

i think the answer is The 1st table represents a function

Lets explain how to solve the problem

- Function is a relation where each input has a single output

- The notation of the function is f(x) = y , where x is the input of the

function and y is the out put of the function

-  The domain of the function is x and the range of the function is y

* Lets find which table represent a function

# First table:

     x      y

    -3      -1

     0      0

    -2      -1

- In the first table

∵ -3 has only -1

∵ 0 has only 0

∵ -2 has only -1

∵ 8 has only 1

∵ Every x has a single y

∴ The table represents a function

∴ The 1st table represents a function

- In the second table

   x       y

   -5       -5

    0       0

   -5       5

    6       -6

∵ x = -5 has y = -5 and y = 5

∴ One x has two values of y

∴ The table doesn't represent a function

- In the third table

    x          y

   -4         8

   -2         2

   -2         4

    0         2

∵ x = -2 has y = 2 and y = 4

∴ One x has two values of y

∴ The table doesn't represent a function

- In the fourth table

    x          y

   -4         2

    3         5

   -4         0

∵ x = -4 has y = 2 and y = 0

∴ One x has two values of y

∴ The table doesn't represent a function

Answer:

It is 12

Step-by-step explanation:

2. Multiply the following:
a. 3.5(x)

Answers

Answer:

3.5x

Step-by-step explanation:

3.5(x)=3.5x

To demonstrate probabilities, a mathematics teacher had students draw cards from a bag which contained 26 red cards and
26 black cards. During class, the bag was dropped, and 5 red cards and 1 black card were lost.
Tell whether the loss of cards changes the probability of drawing a black card from the bag.
If so, was the probability increased or decreased? Support your answer by calculating the probability for each situation.

Answers

Answer:

decreased

Step-by-step explanation:

What is the scale factor from the original triangle to its copy?

.

Original

Copy

Answers

Copy 1 is a scaled copy of the original triangle. The scale factor is 2, because each side in Copy 1 is twice as long as the corresponding side in the original triangle. , , Copy 2 is a scaled copy of the original triangle.

If I had 4 eggs
Leslie gives me 3 eggs
My farm rooster lays 5 eggs
How many eggs do I have?

Answers

Answer:3 eggs

Step-by-step explanation:

You HAD 4 eggs

Leslie gives you 3 eggs

Roosters don’t lay eggs,

Therefore, you have 3 eggs.

I have 12 eggs

Addition is the the process of determining the total sum of two or more numbers. Addition is a basic mathematical operation. The other basic operations are subtraction, multiplication and division.

The number of eggs I have would increase with the number of eggs I am given and the number of eggs the farm rooster lays. This means I would have to add these numbers together.

The number of eggs I have = number of eggs I has initially + number of eggs Leslie gives me + number of eggs the farm rooster lays

4 + 3 + 5 = 12 eggs

A similar question was solved here: https://brainly.com/question/17472003?referrer=searchResults

How many 1 6 cup servings are in 3 4 cups of popcorn? (in simplest fraction form)

Answers

Answer: 8/17

Step-by-step explanation: 16 ÷ 34 = 8/17

Hope this helps ^_^

What is the constant of proportionality in the equation y = 5/9x

Answers

Answer:   5/9

This is assuming the equation is y = (5/9)x

Note it is in the form y = kx with k = 5/9 as the constant of proportionality.

Answer correctly and I will mark you brainless ❤️

Answers

The y intercept is b=7/1000

The mean lifetime of a tire is 48 months with a standard deviation of 77 months. If 147 tires are sampled, what is the probability that the mean of the sample would differ from the population mean by less than 0.83 months?

Answers

Answer:

0.55199

Step-by-step explanation:

When we have a random number of samples, We solve using z score formula

z = (x-μ)/σ/√n where

x is the raw score

μ is the population mean

σ is the population standard deviation

n is random number of samples

z = 0.83/77/√147

z = 0.13069

Probabilty value from Z-Table:

P(x<48.83) = 0.55199

The probability that the mean of the sample would differ from the population mean by less than 0.83 months is 0.55199

The average monthly cost of basic cable TV was $9.73 in 1985 and has increased by about $1.35 each year since then. Let t be the number of years since 1985.
1. Use function notation to write an equation giving the monthly cost of basic cable TV as a function of t.
f(t)=____t+____
2. Using the function from above what is the average monthly cost of basic cable TV for this year?
3. Using the function from 1, what year was the average monthly cost of basic cable TV $23.23?​

Answers

Answer:

$50.23

Step-by-step explanation:

f(t)=1.35(t)+9.73

First I found out how many years are after 1985 so you would do 2020-1985 which equals 30.

I plug it in for t.

f(t)=1.35(30)+9.73

Then you multiply 1.35 and 30 which would leave us with.

f(t)=40.5+9.73

Add them and you get $50.23

Lou deposited $3,500 into a savings account. The simple interest rate is 4%. How much interest will the account earn in 10 years?

Answers

Answer:

2 years : $280

10 years: $1400

Step-by-step explanation:

The amount of interest earned in 10 years will be $1,400.

What is simple interest?

Let P be the principal, R be the rate of interest, and T be the time. Then the interest rate is given as,

The formula for interest is written as,

I = (PRT)/100

$3,500 was put into a savings account Lou. 4% is the simple interest rate.

The amount of interest after 10 years is calculated as,

I = ($3,500 x 4 x 10) / 100

I = $1,400

The amount of interest earned in 10 years will be $1,400.

More about the simple interest link is given below.

https://brainly.com/question/2793278

#SPJ7

What type of number is 739?.
A. Neither
B. Prime
C. Composite ​

Answers

ANSWER

For 739, the answer is: prime anslo written as B

Step-by-step explanation:

739 is a prime number because it has only two distinct divisors: 1 and itself (739).

HAVE A GOOD DAY!

Answer:

its neither

Step-by-step explanation:

Keith wants to plot -8 and -9 and on a number line. Which statement is true? *

Answers

Where the statements at lol

Sarah is training for a bike race. She rides her bike 5 3/4 miles in 1/3 hour. What is Sarah’s rate in miles per hour. Express your answer as a mixed number

Answers

Answer:no

Step-by-step explanation:

Answer:

17 [tex]\frac{1}{4}[/tex]

The length of a rectangle is 97 meters and the width is 14 meters. Find the area. Give your answer without units.
Provide your answer below:

Answers

Use the formula, l•w (length times width)
The length is 97 and the width is 14
97•14=1358

The area of a rectangle is the product of length and width thus the area will be 1358 square meters.

What is a rectangle?

A rectangle is a geometrical figure in which opposite sides are equal.

The angle between any two consecutive sides will be 90 degrees.

The perimeter of the rectangle = 2( length + width).

It is known that,

Area of rectangle = length × width.

Area = 97 x 14 = 1358 sqare meters

Hence "The area of a rectangle is the product of length and width thus the area will be 1358 square meters".

For more about rectangles,

https://brainly.com/question/15019502

#SPJ5

Complete the steps to solve 4.5 ÷ 0.5

Answers

Answer:

9

Step-by-step explanation:

4.5/0.5=9

9*0.5=4.5

hope this helps :3

if it did pls mark brainliest

Answer:

4.5 ÷ 0.5

45/10 ÷ 5/10

45/10 × 10/5

450/50

45/5

= 9

Step-by-step explanation:


Please help me please

Answers

Answer:

Step-by-step explanation:

110_5

Maggie has a 2-gallon pitcher of lemonade. How many liters of lemonade did she make?

I'LL GIVE BRAINLIEST!

Answers

Answer:

2 gallons converted to liters is 7.57 or 8 liters rounded

Maggie made 2 more gallons

Examine the equation. 4(x – 3) = 4x – 12 Which of the following is true? (Check all that apply.) It is a true statement. Any input will result in an equivalent equation. It is equivalent to an equation of the form a = a. It has no solution. Only one input will result in a true statement.

Answers

Answer:

A, B, and C.

Step-by-step explanation:

We have the equation:

[tex]4(x-3)=4x-12[/tex]

Let’s distribute the left. This yields:

[tex]4x-12=4x-12[/tex]

Let’s add 12 to both sides:

[tex]4x=4x[/tex]

And subtract 4x from both sides:

[tex]0=0[/tex]

We get the true statement 0=0.

Hence, A is correct.

Because it is a true statement, any input will yield an equivalent output.

Hence, B is also correct.

We also have the form a=a, where a is 0.

Hence, C is also correct.

Therefore, our correct answers are A, B, and C.

Answer:

It is a true statement.

Any input will result in an equivalent equation.

It is equivalent to an equation of the form a = a.

Solve the equations:

2^(3x−4) = −2

Answers

Answer:

x=NaN your welcome hehe

If h(x) = g[f(x)], use the table of values for f, g, f'and g' to find the value of h'(1).
x f(x) g(x) f'(x) g'(x)
1 3
2
2
6
2 1
00
5
7
3
7
2
7
9
your answer goes here

Answers

Answer:

Hey there :)

h ( x ) = f [ g ( x ) ]

To find h ' ( x )  

We do → f ' [ g ( x ) ] × g ' ( x )  

This is called the chain rule

Given;

g ( 1 ) = 2  →  f ' ( 2 ) = 5

g ' ( 1 ) = 6

Apply the formula

h ' ( 1 ) = f ' [ g ( 1 ) ] × g ' ( 1 )

          = f ' ( 2 ) × 6

          = 5 × 6

h ' ( 1 ) = 30

If h(x) = g[f(x)], the value of h'(1) from the given table is calculated as 18.

To find the value of h'(1), we need to apply the chain rule. The chain rule states that if we have a composite function h(x) = g[f(x)], then its derivative h'(x) is given by [tex]\(h'(x) = g'(f(x)) \cdot f'(x)\)[/tex].

Let's use the table of values to find h'(1):

1. f(1) = 3

2. g(f(1)) = g(3) = 2

3. f'(1) = 2

4. g'(f(1)) = g'(3) = 9

Now, we can find h'(1) using the chain rule:

[tex]\[h'(1) = g'(f(1)) \cdot f'(1) = 9 \cdot 2 = 18.\][/tex]

Therefore, the value of h'(1) is 18.

To know more about value, refer here:

https://brainly.com/question/33648788

#SPJ2

a pump moves 42 gallons in 7 minutes

Answers

Answer:

The correct answer is 6

Step-by-step explanation:

Divide 42 divided by 7 to get 6.

Answer:

What is the question?

Step-by-step explanation:

The pump moves 6 gallons in 1 minute

Divide 42 by 7 = 6

But I don't know the question so I can't tell you the best correct answer.

I just took a guess at what you wanted

PLEASE HELP URGENT WILL GOVE BRAINLIEST

Answers

it is b and there is no explanation

HELP ME PLEASE Find the slope of the line and write it as a rate of change.

Answers

A slope is also known as the gradient of a line. The rate of change is $0.13/ mile.

What is Slope?

A slope also known as the gradient of a line is a number that helps to know both the direction and the steepness of the line.

slope = (y₂-y₁)/(x₂-x₁)

Given two of the points on the line of the function (7000, 900) and (20000, 2600). Now, The rate of change for the given function is the slope of the line as shown in the graph. Therefore, the rate of change will be,

Rate of change = ($2600 - $900)/(20,000 miles - 7,000 miles)

                         = 1700 / 13000

                         = $0.13/ mile

Hence, the rate of change is $0.13/ mile.

Learn more about Slope of Line here:

https://brainly.com/question/14511992

#SPJ2

Simplify each term. please help!! 30 points + brainliest :)
1. -(-3y)³
2. (2c)⁵
3. (-2m)⁴
4. (4g)³
5. (7j)²
6. (-5z)⁵

Answers

Answer:

1.  27y^3

2. 32c^5

3. 16m^4

4. 64g^3

5. 49j^2

6. -3125z^5

Step-by-step explanation:

Can you help answer my question too, please?

Other Questions
Do mountains last forever? A 10 V fan with a current of 2 A is turned on for 30 seconds. Calculate the energy transferred in joules, Andrew spent $45.45 on two shirts that were the same price and a pair of pants. If the pants were $16.99, how much was each shirt? Qu te gusta ms, escuchar msica ____ leer? What is your impression of the difference in milestones in Jonas's society and ours? Explain the drawbacks and benefits of at least two The notebook shows the money Lisa earned and spent on her first day selling kale at theFarmers' Market. A positive number represents money earned. A negative number representsmoney spent. Lisa wants to find her profit for the first day. Which statement best reflects the purpose of satire?A.It seeks to make its readers laugh.B.It tells the personal stories of political officials.C.It reveals flaws in a society in an indirect way.D.It directly criticizes humanity's shortcomings. competition is a competitive advantage based on factors other than price I am writing a school newspaper can someone read it and tell me if it sounds good? Please help me please!Which of the following effects are most closely associated with the turning point described in the excerpt above?a. Founding of Buddhismb. The Age of Explorationc. An increase in the number of hunting and gathering societiesd. The establishment of the first cities Liquid hexane CH3CH24CH3 will react with gaseous oxygen O2 to produce gaseous carbon dioxide CO2 and gaseous water H2O. Suppose 5.2 g of hexane is mixed with 33.0 g of oxygen. Calculate the minimum mass of hexane that could be left over by the chemical reaction. Be sure your answer has the correct number of significant digits. When Jack started his job working for an industrial manufacturing company, he contributed $100at the end of each month into a savings account that earned 1.2 % interest compounded monthlyfor 8 years. At the end of the 8th year, Jack was laid off. To help meet family expenses, Jackwithdrew $285 from the savings account at the end of each month for 2 years. At the end of thesecond year of being unemployed, Jack found another job and started contributing $138 back intothe savings account at the end of each month for the next six years. How much money would hehave in the account at the end of the six years (after returning to work)? Help 10 points please Please help!! Time is almost up and Im having trouble with this question. Giving 20 points HELPPLacey was looking at some data on World Cup soccer matches. She noticed a negative linear relationshipbetween the number of fans attending a match and the number of total goals scored in that match. Here iscomputer output from a least-squares regression analysis for using attendance (in thousands of fans) to predictthe total number of goals scored:PredictorCoefSE CoefTPConstant3.2600.14522.5350.000Attendance (thousands)-0.0090.003-3.3300.001Use this model to predict the total number of goals scored in a match with 90 thousand fans in attendance.You may round your answer to the nearest whole number of goals.goals The author titled this section "Everything Tends to Ruin." What does the word tend mean in this context? Why did the author of this article choose this subtitle for this section? What is "ruined"? Who is doing the "ruining"? HELP NEEDED ASAP I'M GIVING BRAINLIEST1) Select the best answer from the choices below. Light has many properties of waves, but sometimes it also behaves like: aA photon bPolarized light cUltraviolet rays dA stream of particles2) What type of waves transmit signals in satellite communication? aMicrowaves bX-rays cGamma Rays dRadio Waves3) Cell phones use digital signals to send and receive information. These digital signals travel as what kind of wave? aGamma rays bX-rays cMicrowaves dRadio waves 4+ -1 2/3 please help You recently discovered that your job provides a 25% matching towards your retirement.If you deposit $300 each month into a retirement account with an APR of 8.99%, whatwould you type in Excel to find the balance after 30 years? Stars look___ in the country than in the city.more brighter, more bright, brightest, brighter. choose one